use factoring to solve quadratic equation. Check by substitution or by a graphing utility.

4x^2=19x+30​

Answers

Answer 1
Start by moving all terms to left side of equation 4x^2 - 19x - 30 = 0
Multiply 4 x -30 = -120
Now find two factors of 120 that subtract to give 19... 1 x 120, 2 x 60, 3 x 40, 4 x 30, 5 x 24 ✅
[if it had been +120 we would add the factors looking for 19 which in this case don’t exist]
To get -19 we will need 5 + -24
24 is a multiple of 4 so we can use 4 x -6 and 1 x 5
Giving a factored equation of (4x + 5)(x - 6) = 0 [you should check this by expanding to remove the brackets]

One of the brackets must = zero so the solutions are x = -5/4 or x = 6

Some people look at factors of 4 and -30 and then pair then up to find a sum of -19 but this can take longer to finish.

Related Questions

Please help I don’t understand it
I need a reason for both

Answers

9514 1404 393

Answer:

  23) insufficient number of corresponding sides/angles

  24) insufficient number of corresponding sides/angles

Step-by-step explanation:

In general, you need at least 3 corresponding parts to show triangle congruence. At least one of those must be a side. The attachment shows the ways triangles can be declared congruent. (HL works for right triangles, because we know the right angles are congruent to each other.)

__

23) A side is shown congruent (KL ≅ GH), and we know the vertical angles at J are congruent. However, we need another angle before we can declare the ∆GHJ congruent to anything. (One more side doesn't do it, because the two sides would not be on either side of the angle at J.)

  Reason: insufficient data

__

24) A side is shown congruent (MP ≅ QS), and an angle is shown congruent (∠M ≅ ∠Q). As in the previous problem, we need at least one more angle, or we need to know that MN ≅ QR before we can declare the triangles congruent.

  Reason: insufficient data

PLEASE HELP ME PLEASEE

Answers

*☆*――*☆*――*☆*――*☆*――*☆*――*☆*――*☆*――*☆**☆*――*☆*――*☆*――*☆

Answer:  A= 1 b= -8 c= -20

Explanation:

I hope this helped!

<!> Brainliest is appreciated! <!>

- Zack Slocum

*☆*――*☆*――*☆*――*☆*――*☆*――*☆*――*☆*――*☆**☆*――*☆*――*☆*――*☆

Answer:

a = 1

b = -8

c = -20

Step-by-step explanation:

please help me out, god bless you. dont answer with a link or just steal points plz, I really need help

Answers

Answer:

x = 12 radical 2 ithink these is ansewer

20 POINTS!! PLEASE HELP!! GIVING BRAINLIEST:)

Answers

Answer:

d

Step-by-step explanation:

Solve for x over the real numbers:

x^2 + 5 x + 6 = 0

The left hand side factors into a product with two terms:

(x + 2) (x + 3) = 0

Split into two equations:

x + 2 = 0 or x + 3 = 0

Subtract 2 from both sides:

x = -2 or x + 3 = 0

Subtract 3 from both sides:

Answer: |

| x = -2 or x = -3

Use the suggested substitution to write the expression as a trigonometric expression. Simplify your answer as much as possible. Assume 0≤θ≤π2.

[tex]\sqrt{9x^2+36[/tex] , x/2=cot(∅)

Answers

Answer:

[tex]\sqrt{9x^2+36} = 6 \csc(\theta)[/tex]

Step-by-step explanation:

Given

[tex]\frac{x}{2}=\cot(\theta)[/tex]

Required

Express [tex]\sqrt{9x^2+36}[/tex] as trigonometry expression

[tex]\sqrt{9x^2+36}[/tex]

Factorize

[tex]\sqrt{9x^2+36} = \sqrt{9(x^2+4)}[/tex]

Split

[tex]\sqrt{9x^2+36} = \sqrt{9} * \sqrt{(x^2+4)}[/tex]

[tex]\sqrt{9x^2+36} = 3 * \sqrt{(x^2+4)}[/tex]

[tex]\sqrt{9x^2+36} = 3\sqrt{(x^2+4)}[/tex]

We have:

[tex]\frac{x}{2}=\cot(\theta)[/tex]

Make x the subject

[tex]x = 2 \cot(\theta)[/tex]

So:

[tex]\sqrt{9x^2+36} = 3\sqrt{(x^2+4)}[/tex]

[tex]\sqrt{9x^2+36} = 3\sqrt{((2 \cot(\theta))^2+4)}[/tex]

Evaluate all squares

[tex]\sqrt{9x^2+36} = 3\sqrt{4\cot^2(\theta)+4}[/tex]

Factorize

[tex]\sqrt{9x^2+36} = 3\sqrt{4(\cot^2(\theta)+1)}[/tex]

Split

[tex]\sqrt{9x^2+36} = 3\sqrt{4} * \sqrt{\cot^2(\theta)+1}[/tex]

[tex]\sqrt{9x^2+36} = 3*2 * \sqrt{\cot^2(\theta)+1}[/tex]

[tex]\sqrt{9x^2+36} = 6 * \sqrt{\cot^2(\theta)+1}[/tex]

In trigonometry

[tex]\cot^2(\theta)+1 = \csc^2(\theta)[/tex]

So, we have:

[tex]\sqrt{9x^2+36} = 6 * \sqrt{\csc^2(\theta)}[/tex]

Evaluate the square root

[tex]\sqrt{9x^2+36} = 6 * \csc(\theta)[/tex]

[tex]\sqrt{9x^2+36} = 6 \csc(\theta)[/tex]

What is the sum of adding two opposites together? *

Answers

Answer:

I'm guessing the opposites are negative and positive so whenever you add two opposite numbers it will always be 0. The answer will always be 0.

For example:

-3 + 3 = 0

4 - 4 = 0

-15 + 15

Find two consecutive whole numbers that
82 lies between.

Answers

The question as written is incomplete, but square root of 82 is between the whole numbers 9 and 10, whose squares are 81 and 100 respectively.

Answer: The question as written is incomplete, but square root of 82 is between the whole numbers 9 and 10, whose squares are 81 and 100

Pentagon PQRST is shown on the coordinate grid below. Given that the pentagon was translated 6 units to the right and labeled P'Q'R'ST'. what are the coordinates of T'? HE​

Answers

Answer:

i think its X=0

hopes it helps

Step-by-step explanation:

Perform the following subtraction - 1/2 - (- 7/10)

Answers

Answer:

1.2

I t is the answer

please help asap i’m about to give up on this entire assignment i’ve been on this same question for like 20 minutes now and i’ve already asked irl people for help they don’t know either

Answers

Answer:

C and D

Step-by-step explanation:

Area of the top section:

A₁ = x*(x - x + 3) = x*3 = 3x in²

Area of the bottom section:

A₂ = (x + 4)(x - 3) = x² + x - 12 in²

Total area:

A = A₁ + A₂ =      (3x) + (x² + x - 12) in²

Correct choices are C and D

Answer:

C & D

Step-by-step explanation:

top box area = x(x-(x-3)) = 3x

bottom box area = (x+4)(x-3) = x^2 -x -12

so correct ans is C & D

value of x if 3x+2y=11 and 5x-y=14

Answers

Answer:

3x + 2y = 11

3x + y = 11 - 2

3x + y = 9

x + y = 9/3

x + y = 3

x = 3 - y

5x - y = 14

x - y = 14/5

x - y = 2.8

x = 2.8 + y

14Answer:

3

Step-by-step explanation:

solve for y in first equation

5x-y=14

-y=14-5x

y=5x-14

substitute y value (5x-14) into 2nd equation

3x+2 (5x-14)=11

3x+10x-28=11

13x=39

x=3

Someone please help me answer this correctly!!

Answers

Answer:

The answer would be 8 < Rad 70 < 9 .

Step-by-step explanation:

Hope it was helpful & correct :D

-Mojd.

A 12-kg bag of mangoes costs $9.00

Answers

Answer:

I think it would be around $1.33 for each kg

Step-by-step explanation:

You would divide 12 by 9. Once you do that, you get 1.33

Since one refraction cup is half of a cylinder, how could you change the formula for the volume of a cylinder to calculate the volume of a refraction cup?

Answers

Answer:

Although a cylinder is technically not a prism, it shares many of the properties of a prism. Like prisms, the volume is found by multiplying the area of one end of the cylinder (base) by its height.

Step-by-step explanation:

4 less than the product of 8 and a number is equal to 6​

Answers

5 I think that should
6x-4=8
+4 +4
6x=12
6x/2=12/6
X=2

Which steps are needed to solve this equation? 100 points for the first correct answer.

Answers

Answer:

the first one sub 6 from both =15

Step-by-step explanation:

b+6=21

-6 -6

b=15

Can someone pls help. Thx :) I give Brainliest :D

Answers

Answer:

36 flowers are left over

Step-by-step explanation:

54 is 60% of 90

90-54=36

What is the slope of the line passing through the points (-8,5) and (7,0)

Answers

Use the formula y2-y1 over x2-x1
Plug the two points in:
5-0 over (-8)-7 = 5 over (-15) = -3
The slope of the line that passes through points (-8,5) and (7,0) is -3

To solve the problem we must know about the slope of a line.

What is a Slope?

A slope is also known as the gradient of a line is a number that helps to know both the direction and the steepness of the line.

[tex]m = \dfrac{(y_2-y_1)}{(x_2-x_1)}[/tex]

The slope of the line passing through the points (-8,5) and (7,0) is [tex]\dfrac{-1}{3}[/tex].

Given to us

points (-8,5) points (7,0)

Points can be written as,

[tex]y_2 = 0\\y_1 = 5\\x_2=7\\x_1 = -8[/tex]

The slope of a line

We know that the slope of a line whose two points are given is written as,

[tex]m = \dfrac{(y_2-y_1)}{(x_2-x_1)}[/tex]

Substituting the value, we get,

[tex]m = \dfrac{(0-5)}{[7-(-8)]}\\\\m = \dfrac{(0-5)}{[7+8]}\\\\m = \dfrac{(-5)}{[15]}\\\\m= \dfrac{-1}{3}[/tex]

Hence, the slope of the line passing through the points (-8,5) and (7,0) is [tex]\dfrac{-1}{3}[/tex].

Learn more about Slope:

https://brainly.com/question/3605446

Ben Carson was loaned $125.00 to pay $6.00 interest for $100.00 every week ,after 3 weeks how much do he pay back to clear the dept A. $140 B. $147.50 C. $155 D. $135.50 E. 143​

Answers

Answer:

Total amount pay after 3 week = $147.5

Step-by-step explanation:

Given:

Amount of loan taken = $125

Amount interest per $100 = $6 per week

Find:

Total amount pay after 3 week

Computation:

Amount of interest per week = 125[6/100]

Amount of interest per week = $7.5 per week

Amount of interest in three week = 3 x Amount of interest per week

Amount of interest in three week = 3 x 7.5

Amount of interest in three week = $22.5

Total amount pay after 3 week = Amount of loan taken + Amount of interest in three week

Total amount pay after 3 week = 125 + 22.5

Total amount pay after 3 week = $147.5

⚠️URGENT PLS HELP

LINKS WILL BE REPORTED

Answers

Answer:

e and c

Step-by-step explanation:

Can some one help me ASAP and thanks :)

Answers

Step-by-step explanation:

Hi,

For the first one...

Input: 0    Output: 1

Input: 2    Output: 2

Input: -8   Output: -3

Input: 100 Output: 51

And then for the creat your own...

Divide by 5 and add 3

Input: 50   Output: 13

Input: 25   Output: 8

Input: -75  Output: -12

Input: 150  Output: 33

I hope this helps :)

Determine which number is the solution of the inequality: f + 2 < 9

Answers

Answer:

f < 7

Step-by-step explanation:

f can be any number that is less than 7

Answer:

6 and less will give you a number less then 9

Step-by-step explanation:

We know its not 7

because 7 + 2 < 9 = 9 = 9

we want something that is less then 9

1 up to 7 would give you an answer less then 9

6 + 2< 9 = 8 < 9

5 + 2 < 9 = 7 < 9

4 + 2 < 9 = 6 < 9

and more

Please help, brainliest for correct answer

Answers

Answer:

11494.04 units

Step-by-step explanation:

V = 4/3 [tex]\pi[/tex] r³ = 4/3 · [tex]\pi[/tex] · 14³ ≈ 11494.04032

Cameron has a balance of $10,200 on his credit card. He threw the card away so he can never use it again. He has 3.5 years to pay off the balance. The interest rate on his card is 21%.

At the end of the 3.5 years, how much money in interest alone will Cameron have paid?

When Cameron finally pays off his credit card, how much will he have paid in all?

Answers

Answer:

the interest is $9,676.91 and the future value amount is $19,876.91

Step-by-step explanation:

The computation is shown below;

The future value is

= Present value × (1 + rate of interest)^number of years

= $10,200 × (1 + 0.21)^3.5

=  $19,876.91

Now the interest amount is

= Future value - present value

= $19,876.91 - $10,200

= $9,676.91

Hence, the interest is $9,676.91 and the future value amount is $19,876.91

solve the system of equations -3x+2y=-3 and -x-y=-11 by combining the equations

Answers

Answer:

-4x, 1y, -14

Step-by-step explanation:

Hope this helps :)

find the missing length

Answers

Number 1. Is 10

Number 2. G-E is 20 and F-E is 10

24+z/10 = 31 solve for Z

Answers

Z=70

Simplify both sides of the equation
Subtract 24 from both sides
Multiple both sides by 10

The hypotenuse of a right triangle measures 11 cm and one of its legs measures 1 cm. Find the measure of the other leg. If necessary, round to the nearest tenth.

Answers

Answer:

10.9 is the length of the other leg

Step-by-step explanation:

use the reverse Pythagorean therm

the square root of 11(squared) - 1(squared) = 10.9

I NEED HELP PLEASE, THANKS!!!!

Answers

Answer:

9809.223

Step-by-step explanation:

8700 * 1.1274968

9809.223

Factoring out the GCF of the polynomial 2x^6+2x^5 will give

Answers

2x^5(x+1)

hope this helps
Other Questions
NO LINKS I WILL REPORT!!! But help plssss Which of these triangles are definitely not congruent to any of the others?Check all that apply. The Lion, The Witch, and The Wardrobe, Chapter 14At the beginning of chapter 14, Susan and Lucy ahave an important conversation. bcan't sleep. cfight dwarves with their weapons. dmeet handsome princes. Can you help me with these problems and explain step by step Cuz i have no clue how to do them how many moles of CO2 molecules are in 120 g of CO2 Priscilla is driving her car on a busy street and Harvey passes her on his motorcycle. What will happen to the sound from his motorcycle after it passes her car?A. The frequency of the sound will decrease.B. The amplitude of the sound will increase.C. The intensity of the sound will increase.D. The wavelength of the sound will decrease.E. The velocity of the sound will increase. You start at (10, 9). You move left 9 units. Where do you end? Discuss Henry Clay's stance on whether or not to go to war Write te expanded form of 54 72/100 in a fraction form How can you prevent guinea worm infection Find the distance between (-22,15) and (30, -25). Take your time please. Jessica bought 1/2 pound of roast beef and 5/6 pound of ham. Which pairs of fractions are equivalent to the amounts Jessica bought? Mark all that apply.A.8/16 and 10/16B.6/12 and 10/12C.5/10 and 8/10D.12/24 and 20/24 Can Anybody Do This For me Total surface area if 5.5in 4.5in and 3 in A salmon is swimming in a river that is flowing downstream at a speed of 2 miles per hour. The slamon can swim 12 miles upstream in the same amount of time it would take 24 miles downstream. What is the speed of the salmon in still water? The midpoint of the line segment joining P(-2, 3) and Q(4,-1) is M.The point C has coordinates (-1,-2).Show that CM is perpendicular to PQ. Escriba la palabra o frase que mejor completa la oracin.donde podrir a jugar bsquetbolCerca de mi casa estn construyendocon mis amigos.O A. un centro comunitarioB. una cooperativaO C. un negocio localO D. un refugio para personas sin hogar A chocolate bar cost 2 dirham. Alyazia spends 18dh on chocolate bars for her siblings. Write an equation to express how many chocolate bars she bought.pllssss helpppp A mason gets $684 for 9 daysof work how many daysshould he work to get $912 PLs help this is prt 2 for recent question